Question on point set topologyDefinition of Borel setsA “complementary” topologyFinite vs infinite...

Can a wizard cast a spell during their first turn of combat if they initiated combat by releasing a readied spell?

Geography in 3D perspective

Knife as defense against stray dogs

Why is indicated airspeed rather than ground speed used during the takeoff roll?

What are substitutions for coconut in curry?

Does .bashrc contain syntax errors?

How is the partial sum of a geometric sequence calculated?

Help prove this basic trig identity please!

How are passwords stolen from companies if they only store hashes?

Is there a hypothetical scenario that would make Earth uninhabitable for humans, but not for (the majority of) other animals?

Does the attack bonus from a Masterwork weapon stack with the attack bonus from Masterwork ammunition?

Generic TVP tradeoffs?

In what cases must I use 了 and in what cases not?

Is it possible to stack the damage done by the Absorb Elements spell?

Pronounciation of the combination "st" in spanish accents

Print last inputted byte

Could Sinn Fein swing any Brexit vote in Parliament?

Violin - Can double stops be played when the strings are not next to each other?

Loading the leaflet Map in Lightning Web Component

Print a physical multiplication table

두음법칙 - When did North and South diverge in pronunciation of initial ㄹ?

What favor did Moody owe Dumbledore?

How does 取材で訪れた integrate into this sentence?

Describing a chess game in a novel



Question on point set topology


Definition of Borel setsA “complementary” topologyFinite vs infinite distinction in Rudin's AnalysisThe set of rationals in $(0,1)$ is not a $G_delta$Limit point of an infinite subset of a compact setIf $U ⊂ mathbb{R}^n$ is open and $B ⊂ U$, then why is it that $B$ relatively open in $U$ if and only if $B$ is open?Question about Theorem 2.24 in Baby RudinShowing that if closed subsets don't intersect then there exists open sets in which they exist that also don't intersectDifference between closure and closed cover of a setIs there an analogue for a compact set using closed sets?













0












$begingroup$


Does there exist a closed set which is an intersection of a collection of infinite open sets?










share|cite|improve this question







New contributor




Tony Tong is a new contributor to this site. Take care in asking for clarification, commenting, and answering.
Check out our Code of Conduct.







$endgroup$








  • 5




    $begingroup$
    Intersect $(-tfrac{1}{n}, tfrac{1}{n})$ for $n = 1, 2, ldots$ and consider what set you get
    $endgroup$
    – Brevan Ellefsen
    32 mins ago










  • $begingroup$
    Oh it will get ${0}$
    $endgroup$
    – Tony Tong
    27 mins ago
















0












$begingroup$


Does there exist a closed set which is an intersection of a collection of infinite open sets?










share|cite|improve this question







New contributor




Tony Tong is a new contributor to this site. Take care in asking for clarification, commenting, and answering.
Check out our Code of Conduct.







$endgroup$








  • 5




    $begingroup$
    Intersect $(-tfrac{1}{n}, tfrac{1}{n})$ for $n = 1, 2, ldots$ and consider what set you get
    $endgroup$
    – Brevan Ellefsen
    32 mins ago










  • $begingroup$
    Oh it will get ${0}$
    $endgroup$
    – Tony Tong
    27 mins ago














0












0








0





$begingroup$


Does there exist a closed set which is an intersection of a collection of infinite open sets?










share|cite|improve this question







New contributor




Tony Tong is a new contributor to this site. Take care in asking for clarification, commenting, and answering.
Check out our Code of Conduct.







$endgroup$




Does there exist a closed set which is an intersection of a collection of infinite open sets?







analysis






share|cite|improve this question







New contributor




Tony Tong is a new contributor to this site. Take care in asking for clarification, commenting, and answering.
Check out our Code of Conduct.











share|cite|improve this question







New contributor




Tony Tong is a new contributor to this site. Take care in asking for clarification, commenting, and answering.
Check out our Code of Conduct.









share|cite|improve this question




share|cite|improve this question






New contributor




Tony Tong is a new contributor to this site. Take care in asking for clarification, commenting, and answering.
Check out our Code of Conduct.









asked 35 mins ago









Tony TongTony Tong

272




272




New contributor




Tony Tong is a new contributor to this site. Take care in asking for clarification, commenting, and answering.
Check out our Code of Conduct.





New contributor





Tony Tong is a new contributor to this site. Take care in asking for clarification, commenting, and answering.
Check out our Code of Conduct.






Tony Tong is a new contributor to this site. Take care in asking for clarification, commenting, and answering.
Check out our Code of Conduct.








  • 5




    $begingroup$
    Intersect $(-tfrac{1}{n}, tfrac{1}{n})$ for $n = 1, 2, ldots$ and consider what set you get
    $endgroup$
    – Brevan Ellefsen
    32 mins ago










  • $begingroup$
    Oh it will get ${0}$
    $endgroup$
    – Tony Tong
    27 mins ago














  • 5




    $begingroup$
    Intersect $(-tfrac{1}{n}, tfrac{1}{n})$ for $n = 1, 2, ldots$ and consider what set you get
    $endgroup$
    – Brevan Ellefsen
    32 mins ago










  • $begingroup$
    Oh it will get ${0}$
    $endgroup$
    – Tony Tong
    27 mins ago








5




5




$begingroup$
Intersect $(-tfrac{1}{n}, tfrac{1}{n})$ for $n = 1, 2, ldots$ and consider what set you get
$endgroup$
– Brevan Ellefsen
32 mins ago




$begingroup$
Intersect $(-tfrac{1}{n}, tfrac{1}{n})$ for $n = 1, 2, ldots$ and consider what set you get
$endgroup$
– Brevan Ellefsen
32 mins ago












$begingroup$
Oh it will get ${0}$
$endgroup$
– Tony Tong
27 mins ago




$begingroup$
Oh it will get ${0}$
$endgroup$
– Tony Tong
27 mins ago










1 Answer
1






active

oldest

votes


















4












$begingroup$

$$mathbb{R}capmathbb{R}capmathbb{R}capcdots$$






share|cite|improve this answer









$endgroup$













  • $begingroup$
    But R is an open set, the intersection is also R so it is still an open set
    $endgroup$
    – Tony Tong
    31 mins ago








  • 1




    $begingroup$
    And also closed
    $endgroup$
    – Keen-ameteur
    29 mins ago






  • 1




    $begingroup$
    While extremely simple, this example has the unfortunate side effect of also being open, which could further confound the OP who seems to be wondering why the intersection of open sets is not open in general (admittedly, the OP is probably also drawing a false dichotomy between open and closed sets, so I suppose this helps with that)
    $endgroup$
    – Brevan Ellefsen
    25 mins ago












  • $begingroup$
    @BrevanEllefsen: +1, but I couldn't resist... :-)
    $endgroup$
    – parsiad
    25 mins ago













Your Answer





StackExchange.ifUsing("editor", function () {
return StackExchange.using("mathjaxEditing", function () {
StackExchange.MarkdownEditor.creationCallbacks.add(function (editor, postfix) {
StackExchange.mathjaxEditing.prepareWmdForMathJax(editor, postfix, [["$", "$"], ["\\(","\\)"]]);
});
});
}, "mathjax-editing");

StackExchange.ready(function() {
var channelOptions = {
tags: "".split(" "),
id: "69"
};
initTagRenderer("".split(" "), "".split(" "), channelOptions);

StackExchange.using("externalEditor", function() {
// Have to fire editor after snippets, if snippets enabled
if (StackExchange.settings.snippets.snippetsEnabled) {
StackExchange.using("snippets", function() {
createEditor();
});
}
else {
createEditor();
}
});

function createEditor() {
StackExchange.prepareEditor({
heartbeatType: 'answer',
autoActivateHeartbeat: false,
convertImagesToLinks: true,
noModals: true,
showLowRepImageUploadWarning: true,
reputationToPostImages: 10,
bindNavPrevention: true,
postfix: "",
imageUploader: {
brandingHtml: "Powered by u003ca class="icon-imgur-white" href="https://imgur.com/"u003eu003c/au003e",
contentPolicyHtml: "User contributions licensed under u003ca href="https://creativecommons.org/licenses/by-sa/3.0/"u003ecc by-sa 3.0 with attribution requiredu003c/au003e u003ca href="https://stackoverflow.com/legal/content-policy"u003e(content policy)u003c/au003e",
allowUrls: true
},
noCode: true, onDemand: true,
discardSelector: ".discard-answer"
,immediatelyShowMarkdownHelp:true
});


}
});






Tony Tong is a new contributor. Be nice, and check out our Code of Conduct.










draft saved

draft discarded


















StackExchange.ready(
function () {
StackExchange.openid.initPostLogin('.new-post-login', 'https%3a%2f%2fmath.stackexchange.com%2fquestions%2f3152433%2fquestion-on-point-set-topology%23new-answer', 'question_page');
}
);

Post as a guest















Required, but never shown

























1 Answer
1






active

oldest

votes








1 Answer
1






active

oldest

votes









active

oldest

votes






active

oldest

votes









4












$begingroup$

$$mathbb{R}capmathbb{R}capmathbb{R}capcdots$$






share|cite|improve this answer









$endgroup$













  • $begingroup$
    But R is an open set, the intersection is also R so it is still an open set
    $endgroup$
    – Tony Tong
    31 mins ago








  • 1




    $begingroup$
    And also closed
    $endgroup$
    – Keen-ameteur
    29 mins ago






  • 1




    $begingroup$
    While extremely simple, this example has the unfortunate side effect of also being open, which could further confound the OP who seems to be wondering why the intersection of open sets is not open in general (admittedly, the OP is probably also drawing a false dichotomy between open and closed sets, so I suppose this helps with that)
    $endgroup$
    – Brevan Ellefsen
    25 mins ago












  • $begingroup$
    @BrevanEllefsen: +1, but I couldn't resist... :-)
    $endgroup$
    – parsiad
    25 mins ago


















4












$begingroup$

$$mathbb{R}capmathbb{R}capmathbb{R}capcdots$$






share|cite|improve this answer









$endgroup$













  • $begingroup$
    But R is an open set, the intersection is also R so it is still an open set
    $endgroup$
    – Tony Tong
    31 mins ago








  • 1




    $begingroup$
    And also closed
    $endgroup$
    – Keen-ameteur
    29 mins ago






  • 1




    $begingroup$
    While extremely simple, this example has the unfortunate side effect of also being open, which could further confound the OP who seems to be wondering why the intersection of open sets is not open in general (admittedly, the OP is probably also drawing a false dichotomy between open and closed sets, so I suppose this helps with that)
    $endgroup$
    – Brevan Ellefsen
    25 mins ago












  • $begingroup$
    @BrevanEllefsen: +1, but I couldn't resist... :-)
    $endgroup$
    – parsiad
    25 mins ago
















4












4








4





$begingroup$

$$mathbb{R}capmathbb{R}capmathbb{R}capcdots$$






share|cite|improve this answer









$endgroup$



$$mathbb{R}capmathbb{R}capmathbb{R}capcdots$$







share|cite|improve this answer












share|cite|improve this answer



share|cite|improve this answer










answered 32 mins ago









parsiadparsiad

18.4k32453




18.4k32453












  • $begingroup$
    But R is an open set, the intersection is also R so it is still an open set
    $endgroup$
    – Tony Tong
    31 mins ago








  • 1




    $begingroup$
    And also closed
    $endgroup$
    – Keen-ameteur
    29 mins ago






  • 1




    $begingroup$
    While extremely simple, this example has the unfortunate side effect of also being open, which could further confound the OP who seems to be wondering why the intersection of open sets is not open in general (admittedly, the OP is probably also drawing a false dichotomy between open and closed sets, so I suppose this helps with that)
    $endgroup$
    – Brevan Ellefsen
    25 mins ago












  • $begingroup$
    @BrevanEllefsen: +1, but I couldn't resist... :-)
    $endgroup$
    – parsiad
    25 mins ago




















  • $begingroup$
    But R is an open set, the intersection is also R so it is still an open set
    $endgroup$
    – Tony Tong
    31 mins ago








  • 1




    $begingroup$
    And also closed
    $endgroup$
    – Keen-ameteur
    29 mins ago






  • 1




    $begingroup$
    While extremely simple, this example has the unfortunate side effect of also being open, which could further confound the OP who seems to be wondering why the intersection of open sets is not open in general (admittedly, the OP is probably also drawing a false dichotomy between open and closed sets, so I suppose this helps with that)
    $endgroup$
    – Brevan Ellefsen
    25 mins ago












  • $begingroup$
    @BrevanEllefsen: +1, but I couldn't resist... :-)
    $endgroup$
    – parsiad
    25 mins ago


















$begingroup$
But R is an open set, the intersection is also R so it is still an open set
$endgroup$
– Tony Tong
31 mins ago






$begingroup$
But R is an open set, the intersection is also R so it is still an open set
$endgroup$
– Tony Tong
31 mins ago






1




1




$begingroup$
And also closed
$endgroup$
– Keen-ameteur
29 mins ago




$begingroup$
And also closed
$endgroup$
– Keen-ameteur
29 mins ago




1




1




$begingroup$
While extremely simple, this example has the unfortunate side effect of also being open, which could further confound the OP who seems to be wondering why the intersection of open sets is not open in general (admittedly, the OP is probably also drawing a false dichotomy between open and closed sets, so I suppose this helps with that)
$endgroup$
– Brevan Ellefsen
25 mins ago






$begingroup$
While extremely simple, this example has the unfortunate side effect of also being open, which could further confound the OP who seems to be wondering why the intersection of open sets is not open in general (admittedly, the OP is probably also drawing a false dichotomy between open and closed sets, so I suppose this helps with that)
$endgroup$
– Brevan Ellefsen
25 mins ago














$begingroup$
@BrevanEllefsen: +1, but I couldn't resist... :-)
$endgroup$
– parsiad
25 mins ago






$begingroup$
@BrevanEllefsen: +1, but I couldn't resist... :-)
$endgroup$
– parsiad
25 mins ago












Tony Tong is a new contributor. Be nice, and check out our Code of Conduct.










draft saved

draft discarded


















Tony Tong is a new contributor. Be nice, and check out our Code of Conduct.













Tony Tong is a new contributor. Be nice, and check out our Code of Conduct.












Tony Tong is a new contributor. Be nice, and check out our Code of Conduct.
















Thanks for contributing an answer to Mathematics Stack Exchange!


  • Please be sure to answer the question. Provide details and share your research!

But avoid



  • Asking for help, clarification, or responding to other answers.

  • Making statements based on opinion; back them up with references or personal experience.


Use MathJax to format equations. MathJax reference.


To learn more, see our tips on writing great answers.




draft saved


draft discarded














StackExchange.ready(
function () {
StackExchange.openid.initPostLogin('.new-post-login', 'https%3a%2f%2fmath.stackexchange.com%2fquestions%2f3152433%2fquestion-on-point-set-topology%23new-answer', 'question_page');
}
);

Post as a guest















Required, but never shown





















































Required, but never shown














Required, but never shown












Required, but never shown







Required, but never shown

































Required, but never shown














Required, but never shown












Required, but never shown







Required, but never shown







Popular posts from this blog

Gersau Kjelder | Navigasjonsmeny46°59′0″N 8°31′0″E46°59′0″N...

What is the “three and three hundred thousand syndrome”?Who wrote the book Arena?What five creatures were...

Are all UTXOs locked by an address spent in a transaction?UTXO all sent to change address?Signing...